Thomas: The club president had no right to disallow Jeffrey's vote. Club rules say that only members in good stan...

Lucie on November 5, 2019

Correct Answer A

Hello, Could you please explain why A is the correct answer? Thank you!

Reply
Create a free account to read and take part in forum discussions.

Already have an account? log in

Annie on November 5, 2019

Hi @Lucie,

This question is asking you to find the flaw in Thomas' argument. The helpful part of the question though is that Althea has helped point you to that flaw. Thus, the test is giving us two examples of faulty logic, and you're looking for the answer choice that demonstrates where the fault is. Let's outline the arguments:

Thomas:
Premise: Club rules say that only members in good standing may vote.
Premise: All members whose dues are fully paid are members in good standing.
Premise: Jeffrey has always paid his dues on time.
Conclusion: The club president has no right to disallow Jeffrey's vote.

Althea:
Premise: My two-year old niece is a citizen.
Premise: Only citizens can legally vote.
Conclusion: My two- year-old niece can vote.

The key to both arguments is the word "only." While the premises tells us that only club members in good standing can vote, they do not tell us that there is not another requirement. For instance, it could be to vote you must be both a member in good standing and you must live in a certain town to vote. Althea points out this flaw, as her niece may be a citizen, but she is not 18 years old (assuming they're in the US) and therefore can't vote.

Answer Choices:
(A) is correct. As described above, the premises show that Jeffrey is not prohibited from voting for the specific reason that he is a member in good standing. However, they do not indicate that he actually is allowed to vote as there may be other reasons he is not allowed to.

(B) is incorrect. There is no discussion of the character of the club president.

(C) is incorrect. There is no discussion of someone denying a statement.

(D) is incorrect. The reason for the vote is irrelevant to Jeffrey's ability to vote.

(E) is incorrect. The question is asking about Thomas' argument, not Althea's, so this answer choice can easily be dismissed.